Laplace Transform solution help

Click For Summary
The discussion focuses on solving an initial value problem using the Laplace Transform, specifically the equation y'' + 4y = u(pi(t)) - u(3pi(t)) with initial conditions y(0) = 7 and y'(0) = 5. The user successfully derives the Laplace transform and combines terms, resulting in Y expressed in terms of s. However, they struggle with finding the inverse Laplace transform of the first term involving exponentials and seek guidance on handling this complexity. They realize that partial fractions may be necessary for solving the problem, particularly in relation to the step function. The conversation highlights the challenges of applying Laplace transforms with exponential terms and the need for a deeper understanding of the relevant theorems.
Melawrghk
Messages
140
Reaction score
0

Homework Statement


Find the solution of the given initial value problem:
y''+4y=upi(t)-u3pi(t) y(0)=7, y'(0)=5

The Attempt at a Solution


So I found the L{} of the above equation:
s2Y-s*f(0)-f'(0)+4Y = (e-pi*s)/s-(e-3pi*s)/s

Combining and substituting the numbers I get:
Y=\frac{e^{-pi*s}-e^{-3pi*s}}{s(s^{2}+4)}+\frac{6s+3}{s^2+4}

I know how to do the second term's inverse Laplace, but not the first. Here is what I tried:
I can see that I can't get rid of the exponentials in any way other than using the step function again. And the other denominator factor (s^2+4) can be potentially used to get sine. So that:
Y=\frac{1}{2}(\frac{e^{-pi*s}-e^{-3pi*s}}{s})\frac{2}{s^{2}+4}

This is where I don't know what to do. I can't separate them and I don't know of a way to do Laplace inverse of a product.

Any help would be really appreciated. Thanks
 
Last edited:
Physics news on Phys.org
partial fractions

Also, the exponentials should be powers of s not t.
 
How do I do partial fractions with exponentials? Do I use like Ae^(-pi*s) instead of the usual A?

Edit: I fixed the powers
 
Last edited:
Nevermind partial fractions won't help here. You need to have proven a theorem about the laplace transform of the step function times another function.
 
Nah, I got it, you do need partial fractions. Thanks.
 
Question: A clock's minute hand has length 4 and its hour hand has length 3. What is the distance between the tips at the moment when it is increasing most rapidly?(Putnam Exam Question) Answer: Making assumption that both the hands moves at constant angular velocities, the answer is ## \sqrt{7} .## But don't you think this assumption is somewhat doubtful and wrong?

Similar threads

  • · Replies 1 ·
Replies
1
Views
1K
  • · Replies 10 ·
Replies
10
Views
2K
  • · Replies 7 ·
Replies
7
Views
2K
  • · Replies 4 ·
Replies
4
Views
3K
  • · Replies 1 ·
Replies
1
Views
928
Replies
9
Views
2K
  • · Replies 3 ·
Replies
3
Views
3K
Replies
3
Views
2K
  • · Replies 1 ·
Replies
1
Views
995
Replies
2
Views
2K